What is -6x multiplied by y

Answers

Answer 1

✿ ✿ ✿ ✿ ✿ [tex]Hi! :)[/tex] ✶ ✶ ✶ ✶ ✶ ✶ ✶ ✶ ✶ ✶ ✶ ✶ ✶ ✶ ✶ ✶ ✶ ✶ ✶ ✶ ✶ ✶ ✶ ✶ ✶ ✶ ✶

[tex]QUESTION:[/tex]

What is -6x multiplied by y?    ✱  ✱  ✱  ✱  ✱

[tex]ANSWER:[/tex]  

[tex]-6xy[/tex]

Hope this helps you!

Good luck on your assignment & enjoy your day!

[tex]GraceRosalia[/tex]

~Just an emotional teen who listens to music


Related Questions

16.-:- En una granja hay 52 animales, entre vacas y chanchos. Si se sabe que la
frecuencia relativa de vacas es 14, ¿Cuántos chanchos tiene la granja? *
10
13
38
39

Answers

Answer:

what it mean???? I'm just asking

Q16. Find x^2 + 1/x^2 , if x + 1/x = 3

Answers

Answer:

[tex] 2 \times x \times \frac{1}{x} = 2 \\ {x}^{2} + {y }^{2} + 2xy = {(x + y)}^{2} \\ {x}^{2} + \frac{1}{ {x}^{2} } + 2 = {(x + \frac{1}{x} })^{2} = {3}^{2} = 9[/tex]

If -13(q+4) is equal to 7q+16 what’s is q

Answers

Answer:

q = - 3.4

Step-by-step explanation:

- 13(q + 4) = 7q + 16 ← distribute parenthesis on left side

- 13q - 52 = 7q + 16 ( subtract 7q from both sides )

- 20q - 52 = 16 ( add 52 to both sides )

- 20q = 68 ( divide both sides by - 20 )

q = - 3.4

Simplify the following:

1)y^6/y^2
2)b^7/b^5

Answers

Answer:

look at the photo i have sent

The teacher-student ratio at Dunn Middle School is 1 to 23. If there are 966 students, how many
teachers are there?

Answers

Answer:

42 teachers

Explanation:

Teacher to student ratio is 1:23

If there are 966 students, then divide by 23 to get how many teachers their are.

966/23=42

A 145​-inch board is cut into two pieces. One piece is four times the length of the other. Find the length of the shorter piece.
i need to figure this out in the next 20 mins cause i have to turn this in to get a good grade please...

Answers

Answer:

4x + x = 145,                            x=29 (shorter piece)

5x=145

x=145/5 = 29

Step-by-step explanation:

For any nonnegative real number.

Answers

Answer:

A

Step-by-step explanation:

We know that [tex]\sqrt{b}=b^{1/2}[/tex].

We also know that [tex](a^b)^c=a^{bc}[/tex].

We can combine these facts to get [tex](\sqrt{b})^2=(b^{1/2})^2=b^{1/2*2}=b^{1}=b[/tex].

So, the answer is [tex]\boxed{A}[/tex] and we're done!

Note: the non-negative part or the equation comes from the fact that you assume the square root of a negative number doesn't exist. You will change this assumption later in the year, or in further years.

PLZZZ HELPPPP ILL MARK BRAINLIEST TO FIRST AND CORRECT ANSWER THAT HAS WORK AND A THX TO SECOND AND CORRECT ANSWER.

What is twice the sum of -1 1/2 and 0.04

A. -2.92
B. -1.2
C. -1.5
D. -2.5

Answers

-1 1/2= -1.5
0.04=0.04

-1.5+0.04=-1.46
-1.46*2=-2.92

A
A. -2.92
To get this answer you add the numbers to find the sum. When you get the sum, multiply it by 2. You will get -2.92

help please !!:))))))​

Answers

Point A = (-1,-1) point B= (2,-1)

-1 2\5 + 4\9 expression equilvant plzz fast

Answers

Answer:        -0.9555556

Step-by-step explanation:

Mrs. Peña is taking a group of 50 students on a field trip to the Georgia Aquarium. If 20 of the students are boys, what percent of the students are boys?

Answers

Answer:

40%

Step-by-step explanation:

20/50 = 40/100 = 40%

Consider the graph of f(x).
12
10
-6
- 4
120
2 4 6 8
Which statements are true about tthe function represented by the graph?
Select all that apply.
The y-interceptis (0.-2).
The x-interceptis (0,5).
The domain of the function is (-3.3)
The domain of the function is all real numbers.

Answers

Answer:

The domain of the function is all real numbers.

Step-by-step explanation:

The answer is not A because the y-axis is the vertical (straight up and down) axis, and it crosses of that at positive 5.

The answer is not B because the x-axis is the horizontal axis, and it crosses that at negative 2.

Don't let them confuse you by saying the intercepts of one is the other!

It is also not C because the answer is D, we know this because the arrows at each end of the arrow signify that the line keeps going infinitely in each direction.

Hope this helps!

I dont know this no links though

Answers

Answer:

ok do the sqare root of 54 and 24 round to the nearest tenth then multiply the 54 by 3 and the 24 by 4 then subtract them too

Step-by-step explanation:

Please help me with this homework

Answers

Answer:

2-0

_____

-8-(-1)

explanation:

help please and thank youu

Answers

Answer:

5

Step-by-step explanation:

f(x) just means y

find the spot on the line that hits y at 7.

then look directly down to the x line to see that the intersection is (5,7)

Use the probability distribution to complete parts (a) and (b) below.
The number of school-related extracurricular activities per student
Activities 0 1 2 3 4 5 6 7
Probability 0.059 0.121 0.164 0.179 0.212 0.127 0.086 0.052
(a) Find the mean, variance, and standard deviation of the probability distribution.
The mean is 3.3
(Round to one decimal place as needed.)
The variance is
(Round to one decimal place as needed.)

Answers

this could take a while to think about but uhh i dont know,

The variance of the probability distribution is 3.4.

To find the variance of the probability distribution, calculate the following:

1. The squared deviation of each activity value from the mean.

2. The product of each squared deviation and its corresponding probability.

3. The sum of these products.

1. Calculate the squared deviation of each activity value from the mean (3.3):

[tex]\[ (0 - 3.3)^2 = 3.3^2 = 10.89 \]\\[/tex]

[tex]\[ (1 - 3.3)^2 = (-2.3)^2 = 5.29 \][/tex]

[tex]\[ (2 - 3.3)^2 = (-1.3)^2 = 1.69 \][/tex]

[tex]\[ (3 - 3.3)^2 = (-0.3)^2 = 0.09 \][/tex]

[tex]\[ (4 - 3.3)^2 = (0.7)^2 = 0.49 \][/tex]

[tex]\[ (5 - 3.3)^2 = (1.7)^2 = 2.89 \][/tex]

[tex]\[ (6 - 3.3)^2 = (2.7)^2 = 7.29 \][/tex]

[tex]\[ (7 - 3.3)^2 = (3.7)^2 = 13.69 \][/tex]

2. the product of each squared deviation and its corresponding probability:

[tex]\[ 0.059 \times 10.89 = 0.64351 \][/tex]

[tex]\[ 0.121 \times 5.29 = 0.63909 \][/tex]

[tex]\[ 0.164 \times 1.69 = 0.27716 \][/tex]

[tex]\[ 0.179 \times 0.09 = 0.01611 \][/tex]

[tex]\[ 0.212 \times 0.49 = 0.10388 \][/tex]

[tex]\[ 0.127 \times 2.89 = 0.36703 \][/tex]

[tex]\[ 0.086 \times 7.29 = 0.62814 \][/tex]

[tex]\[ 0.052 \times 13.69 = 0.71188 \][/tex]

3. Sum up these products to get the variance:

[tex]\[ \text{Variance} = 0.64351 + 0.63909 + 0.27716 + 0.01611 + 0.10388 + 0.36703 + 0.62814 + 0.71188[/tex]

              = [tex]3.38680 \][/tex]

              =3.4

So, the variance is 3.4.

Learn more about variance here:

https://brainly.com/question/29253308

#SPJ4

3/2(4x – 1) – 3x = 5/4 – (x + 2)

Answers

X = 3/16 or in decimal form X = 0.1875

Hope it helps!

Brainliest would be nice but you don’t have to :)

Last week, Alan's Diner sold 10 milkshakes with whipped cream on top and 15 milkshakes without whipped cream. What percentage of the milkshakes had whipped cream?

Answers

Answer:

4

Step-by-step explanation:

Answer:

4 milkshakes with whipped cream

Round 38,451 to the nearest..
Ten thousand:
Thousand:
Hundred:
Ren:

Answers

Answer:

Ten Thousand: 40000

Thousand: 38000

Hundred: 38500

Ren: what is ren-?

Step-by-step explanation:

Ten thousand: 40,000

Thousand: 38,000

Hundred: 38,500

Ten: 38,450

When you round you have to look to the number to the right of the number your rounding.

For example, to round 38,451 to the nearest ten thousand you find the ten thousand place (3) and look to the right or it and find that number (8). It the number to the right is 5 or more you round up. If the number is less then 5 it stays the same.

Just try to remember this…

5 or more, raise the score

4 or less let it rest.

8 is greater than 5 so the 3 turns to a 4 and you make everything behind the 4 zeros.

Now let’s try 38,451 rounded to the thousand. Look to the right and see that the number is 4. 4 or less let it rest, so the thousands place (8) stays the same. But don’t forget to make everything behind it zero

Hope this helped!

Could you mark brainliest please?

A rectangular park 60 yards long and 50 yards wide. Plans call for increasing both the length and the width by x yards.
a. Write a function to find the new length of the park.
b. Write a function to find the new width of the park.
c. Write a function to find the area of the park after increasing both the length and width.

Answers

Answer:

C

Step-by-step explanation:

If they are increasing w and l, they need a function for both.

|3 - 5 + 7|.

O 15
O 5
O -15
O -5​

Answers

Answer:

5

Step-by-step explanation:

The answer is 5

because 3 - 5 = -2

and -2 + 7 = 5

so the answer is 5

In your own words, describe how you would use construction tools to construct a perpendicular to a line. Explain as if you were teaching a Geometry newbie. Use vocabulary in your explanation. Give an example of how you could then prove the line you created was perpendicular to the original line.

Answers

Can u do it by yourself

Help
Meeeeeeeeeeeeeeeeeeeeeeeeeeeeeeeeeeeeeeeeeeeeeeeeeeee

Answers

c definitely c hope this helps

what is the GCF of 30 42​

Answers

Answer:

The answer is 6 :)

Step-by-step explanation:

The GCF of two non-zero integers, x(30) and y(42), is the greatest positive integer m(6) that divides both x(30) and y(42) without any remainder.

Answer:GCF=2

Step-by-step explanation:

The lowest factor that fits is 2.

2x15=30

2x21=41

Which number equals 3/4 to the power of negative 2?

Answers

Hi,

I just worked it out and got 16/9

Hope this helps !
16/9
-4*-4=16
-3*-3=9
This should be correct

There are 6 players on a volleyball team. How many players are in a game with 2 teams?

Answers

Answer:

There are 12 players

Step-by-step explanation:

6 x 2 = 12

What do you add to
2 1/5 to make 5

Answers

Answer:

Another 2 1/5

Step-by-step explanation:

2.5 + 2.5 = 5.0 (You don't have to put the .0)

Another example would be:

5.5 + 5.5 = 11.0 (Again, you do not have to put the .0)

- Hope this helps (:

PLEASE HELP IM STUCK ON THIS ONE ​

Answers

Answer:

1. 1/200

2. 15/2

3. 11/36

4. 8/5

Step-by-step explanation:

Insert parentheses and brackets to make4x 8 - 4 + 1 = 12

Answers

Answer:

3

Step-by-step explanation:

number 3.) solve for x

Answers

Answer: D) 9

x = 9........

Other Questions
A 26 ft ladder leaning against a wall begins to slide. How fast is the top of the ladder sliding down the wall at the instant of time when the bottom of the ladder is 10 ft from the wall and sliding away from the wall at the rate of 3 ft/sec? Either your mom or dad ____ us to the gameTake or takes 3.7x10^45 molecules of hydrogen gas = how many grams of hydrogen gas The Goldfish:Write a Narrative from the POV of the Goldfish in "What of this Goldfish would you wish?"Detail the feelings of the emotions of the fish, his opinion of both characters, and what the fishdoes at the end of the story? will mark brainlistWhat was the purpose of the Proclamation of 1763?to force colonists to trade with Britainto stop American colonists from settling in the Midwestto close the harbor in Bostonto force colonists to pay for protection from British soldiers Which image represents a disadvantage of living near the sea?Three men carrying nets into the sea.An outrigger sitting on the sand.A house that has collapsed into the sea. 3 plus four times a number. Help solve ONLY 8 AND 9 plzplz answer this question How did the colonists address the problem of the Intolerable Acts? * what is [tex]8x10 {}^{6} [/tex]what is the equal to Which of the following is the best description of the Declaration of Independence? Where and when did the Renaissance emerge? A student estimated the sum7.95 + 8.11 + 78.5 + 8.05 + 79.4 + 0.815 as:All the numbers begin with a 7 or 8, so use cluster estimation.8 + 8 + 80 + 8 + 80 + 0.8 = 184.8Explain why this reasoning is not accurate. Which ordered pair is a solution to the following:Y= -2x + 5Choose 1 answer:A: Only (2,-9)B: Only (-2,9)C: bothD: neither one coil is placed on top of another. the bottom coil is connected in series to a battery and a switch. with the switch closed, there is a clockwise current in the bottom coil. when the switch is opened, the current in the bottom coil decreases abruptly to zero. what is the direction of the induced current in the top coil, as seen from above while the current in the bottom coil decreases? Perry, Maria, and Lorna are painting rooms in a college dormitory. Working alone, Perry can paint a standard room in 3 hours, Maria can paint a standard room in 2 hours, and Lorna can paint a standard room in 2 hours 30 minutes. Perry, Maria, and Lorna have decided that, to speed up the work, 2 of them will paint a standard room together. Select the value closest to the shortest time in which a 2-person team could paint a standard room, and select the value closest to the longest time in which a 2-person team could paint a standard room, with each person working at his or her respective rate. Make only two selections, one in each column.Shortest time Longest time 49 minutes 1 hour 7 minutes 1 hour 12 minutes 1 hour 14 minutes 1 hour 22 minutes 1 hour 45 minutes how might people adapt to life in an area with steep mountain? The ____ of the ratio of two proportional qualities? Why doesnt an orbiting satellite fall back to earth? mAn African bull elephant has a mass of about 5,5 x 10 grams. A flea has a mass of about 10 x 10 grams. Which statement is true?O A The mass of the elephant is about 55,000,000 times greater than the mass of the flea.OB. The mass of the elephant is about 550,000 times greater than the mass of the flea.O c The mass of the elephant is about 55,000 times greater than the mass of the flea.OD The mass of the elephant is about 5,500,000 times greater than the mass of the flea.